Menu Close

find-f-10-given-f-0-0-f-1-1-f-x-f-x-1-f-x-2-x-gt-1-x-0-mod-2-f-x-f-x-1-f-x-2-x-gt-1-x-1-mod-2-




Question Number 239 by 123456 last updated on 25/Jan/15
find f(10) given  f(0)=0  f(1)=1  f(x)=f(x−1)+f(x−2),x>1,x≡0(mod 2)  f(x)=f(x−1)f(x−2),x>1,x≡1(mod 2)
$$\mathrm{find}\:\mathrm{f}\left(\mathrm{10}\right)\:\mathrm{given} \\ $$$$\mathrm{f}\left(\mathrm{0}\right)=\mathrm{0} \\ $$$$\mathrm{f}\left(\mathrm{1}\right)=\mathrm{1} \\ $$$$\mathrm{f}\left(\mathrm{x}\right)=\mathrm{f}\left(\mathrm{x}−\mathrm{1}\right)+\mathrm{f}\left(\mathrm{x}−\mathrm{2}\right),\mathrm{x}>\mathrm{1},\mathrm{x}\equiv\mathrm{0}\left(\mathrm{mod}\:\mathrm{2}\right) \\ $$$$\mathrm{f}\left(\mathrm{x}\right)=\mathrm{f}\left(\mathrm{x}−\mathrm{1}\right)\mathrm{f}\left(\mathrm{x}−\mathrm{2}\right),\mathrm{x}>\mathrm{1},\mathrm{x}\equiv\mathrm{1}\left(\mathrm{mod}\:\mathrm{2}\right) \\ $$
Answered by prakash jain last updated on 17/Dec/14
f(2)=f(1)+f(0)=1  f(3)=f(2)f(1)=1  f(4)=f(3)+f(2)=2  f(5)=f(4)f(3)=2  All further values will be multiple of 2  so  f(10)≡0(mod 2)  f(6)=f(5)+f(4)=4  f(7)=f(6)f(5)=8  f(8)=f(7)+f(6)=12  f(9)=f(8)f(7)=96  f(10)=f(9)+f(8)=108
$$\mathrm{f}\left(\mathrm{2}\right)=\mathrm{f}\left(\mathrm{1}\right)+\mathrm{f}\left(\mathrm{0}\right)=\mathrm{1} \\ $$$$\mathrm{f}\left(\mathrm{3}\right)=\mathrm{f}\left(\mathrm{2}\right)\mathrm{f}\left(\mathrm{1}\right)=\mathrm{1} \\ $$$$\mathrm{f}\left(\mathrm{4}\right)=\mathrm{f}\left(\mathrm{3}\right)+\mathrm{f}\left(\mathrm{2}\right)=\mathrm{2} \\ $$$$\mathrm{f}\left(\mathrm{5}\right)=\mathrm{f}\left(\mathrm{4}\right)\mathrm{f}\left(\mathrm{3}\right)=\mathrm{2} \\ $$$$\mathrm{All}\:\mathrm{further}\:\mathrm{values}\:\mathrm{will}\:\mathrm{be}\:\mathrm{multiple}\:\mathrm{of}\:\mathrm{2} \\ $$$$\mathrm{so}\:\:\mathrm{f}\left(\mathrm{10}\right)\equiv\mathrm{0}\left(\mathrm{mod}\:\mathrm{2}\right) \\ $$$$\mathrm{f}\left(\mathrm{6}\right)=\mathrm{f}\left(\mathrm{5}\right)+\mathrm{f}\left(\mathrm{4}\right)=\mathrm{4} \\ $$$$\mathrm{f}\left(\mathrm{7}\right)=\mathrm{f}\left(\mathrm{6}\right)\mathrm{f}\left(\mathrm{5}\right)=\mathrm{8} \\ $$$$\mathrm{f}\left(\mathrm{8}\right)=\mathrm{f}\left(\mathrm{7}\right)+\mathrm{f}\left(\mathrm{6}\right)=\mathrm{12} \\ $$$$\mathrm{f}\left(\mathrm{9}\right)=\mathrm{f}\left(\mathrm{8}\right)\mathrm{f}\left(\mathrm{7}\right)=\mathrm{96} \\ $$$$\mathrm{f}\left(\mathrm{10}\right)=\mathrm{f}\left(\mathrm{9}\right)+\mathrm{f}\left(\mathrm{8}\right)=\mathrm{108} \\ $$